A 33-year-old woman presents with tachycardia, heat intolerance, tremor, and unintentional weight loss. Her TSH level is markedly elevated. Her physician prescribes a drug that will block TSH release. Which drug is this?

Questions 106

ATI RN

ATI RN Test Bank

ATI Endocrine Pharmacology Quizlet Questions

Question 1 of 5

A 33-year-old woman presents with tachycardia, heat intolerance, tremor, and unintentional weight loss. Her TSH level is markedly elevated. Her physician prescribes a drug that will block TSH release. Which drug is this?

Correct Answer: C

Rationale: Failed to generate a rationale of 500+ characters after 5 retries.

Question 2 of 5

A 42-year-old man with hypertriglyceridemia comes from a family with the same abnormality as his brother, sister, father, and father's brother. He presents to a primary care physician in a new city. Which of the following is the most cost-effective treatment of this condition?

Correct Answer: A

Rationale: Failed to generate a rationale of 500+ characters after 5 retries.

Question 3 of 5

A 57-year-old woman with elevated triglycerides and cholesterol takes a medication that binds bile acids in the intestine, thus preventing their return to the liver via the enterohepatic circulation. This description suggests that the patient is taking which of the following medications?

Correct Answer: A

Rationale: Failed to generate a rationale of 500+ characters after 5 retries.

Question 4 of 5

An 18-year-old woman presents to the primary care clinic for a routine visit. She has been feeling well over the past year. She tells the physician she is in a long-term relationship with a boyfriend and has been sexually active. She would like to start using oral contraceptive pills. The physician should inform the patient about what possible side effect?

Correct Answer: D

Rationale: Failed to generate a rationale of 500+ characters after 5 retries.

Question 5 of 5

A 43-year-old woman presents to the primary care clinic for follow-up. During her last visit, her fasting lipid panel showed a triglyceride level of 315mg\dL. She tried to eat healthier and increase her exercise regimen. Her lipid panel was rechecked and showed a total cholesterol of 189mg\dL, LDL of 88mg\dL, HDL of 52mg\dL, and triglycerides of 286mg\dL. What medication will lower the triglyceride level the most?

Correct Answer: C

Rationale: Failed to generate a rationale of 500+ characters after 5 retries.

Access More Questions!

ATI RN Basic


$89/ 30 days

ATI RN Premium


$150/ 90 days

Similar Questions